Generate this sequence more efficiently












4












$begingroup$


Is there a more effecient way to generate the sequence shown below.



createOrder[n_] := 
Which[OddQ[n],
Join[Table[2 i - 1, {i, 1, (n + 1)/2}],Reverse@Table[2 i, {i, 1, (n + 1)/2}]],
EvenQ[n],
Join[Table[2 i - 1, {i, 1, n/2 + 1}],Reverse@Table[2 i, {i, 1, n/2}]]]


createOrder[#] & /@ Range[8] // MatrixForm


table










share|improve this question











$endgroup$

















    4












    $begingroup$


    Is there a more effecient way to generate the sequence shown below.



    createOrder[n_] := 
    Which[OddQ[n],
    Join[Table[2 i - 1, {i, 1, (n + 1)/2}],Reverse@Table[2 i, {i, 1, (n + 1)/2}]],
    EvenQ[n],
    Join[Table[2 i - 1, {i, 1, n/2 + 1}],Reverse@Table[2 i, {i, 1, n/2}]]]


    createOrder[#] & /@ Range[8] // MatrixForm


    table










    share|improve this question











    $endgroup$















      4












      4








      4





      $begingroup$


      Is there a more effecient way to generate the sequence shown below.



      createOrder[n_] := 
      Which[OddQ[n],
      Join[Table[2 i - 1, {i, 1, (n + 1)/2}],Reverse@Table[2 i, {i, 1, (n + 1)/2}]],
      EvenQ[n],
      Join[Table[2 i - 1, {i, 1, n/2 + 1}],Reverse@Table[2 i, {i, 1, n/2}]]]


      createOrder[#] & /@ Range[8] // MatrixForm


      table










      share|improve this question











      $endgroup$




      Is there a more effecient way to generate the sequence shown below.



      createOrder[n_] := 
      Which[OddQ[n],
      Join[Table[2 i - 1, {i, 1, (n + 1)/2}],Reverse@Table[2 i, {i, 1, (n + 1)/2}]],
      EvenQ[n],
      Join[Table[2 i - 1, {i, 1, n/2 + 1}],Reverse@Table[2 i, {i, 1, n/2}]]]


      createOrder[#] & /@ Range[8] // MatrixForm


      table







      list-manipulation table sequence






      share|improve this question















      share|improve this question













      share|improve this question




      share|improve this question








      edited 1 hour ago









      Henrik Schumacher

      50.5k469144




      50.5k469144










      asked 4 hours ago









      Hubble07Hubble07

      2,986721




      2,986721






















          3 Answers
          3






          active

          oldest

          votes


















          6












          $begingroup$

          ClearAll[f]
          f[n_Integer] := Join[Range[1, #, 2], Reverse[Range[2, #, 2]]] & /@ Range[2, n];
          TeXForm @ MatrixForm @ f[8]



          $left(
          begin{array}{c}
          {1,2} \
          {1,3,2} \
          {1,3,4,2} \
          {1,3,5,4,2} \
          {1,3,5,6,4,2} \
          {1,3,5,7,6,4,2} \
          {1,3,5,7,8,6,4,2} \
          end{array}
          right)$




          Also



          ClearAll[f2, f3]
          f2[n_Integer] := SortBy[Range@#, {EvenQ, -# (-1 )^Mod[#, 2] &}] & /@ Range[2, n]
          f3[n_] := Ordering[Transpose[{-Mod[#, 2], -# (-1 )^Mod[#, 2]} &@Range[#]]] & /@ Range[2, n]

          f[8] == f2[8] == f3[8]



          True







          share|improve this answer











          $endgroup$





















            5












            $begingroup$

               fGetList[n_]:= (Select[Range[#], OddQ]~Join~Reverse@Select[Range[#], EvenQ]) & /@Range[n] // Rest

            fGetList[10] // MatrixForm // TeXForm


            $
            left(
            begin{array}{c}
            {1,2} \
            {1,3,2} \
            {1,3,4,2} \
            {1,3,5,4,2} \
            {1,3,5,6,4,2} \
            {1,3,5,7,6,4,2} \
            {1,3,5,7,8,6,4,2} \
            {1,3,5,7,9,8,6,4,2} \
            {1,3,5,7,9,10,8,6,4,2} \
            end{array}
            right)$



            another version



            fGetList2[n_?IntegerQ] := 
            Flatten@MapAt[Reverse, GatherBy[Range[#], OddQ], 2] & /@ Range[2, n]

            fGetList2[10] // MatrixForm // TeXForm


            $left(
            begin{array}{c}
            {1,2} \
            {1,3,2} \
            {1,3,4,2} \
            {1,3,5,4,2} \
            {1,3,5,6,4,2} \
            {1,3,5,7,6,4,2} \
            {1,3,5,7,8,6,4,2} \
            {1,3,5,7,9,8,6,4,2} \
            {1,3,5,7,9,10,8,6,4,2} \
            end{array}
            right)$






            share|improve this answer











            $endgroup$





















              2












              $begingroup$

              cg = Compile[{{a, _Integer, 1}, {b, _Integer, 1}, {i, _Integer}},
              Join[a[[1 ;; Quotient[i + 1, 2]]], b[[-Quotient[i, 2] ;; -1]]],
              CompilationTarget -> "WVM",
              RuntimeAttributes -> {Listable},
              Parallelization -> True
              ];
              g[n_Integer] := cg[Range[1, n + 1, 2], Range[n + Mod[n, 2], 2, -2], Range[2, n + 1]];





              share|improve this answer











              $endgroup$













              • $begingroup$
                there's a little bug with g[n_?Integer], use g[n_Integer] or g[n_?IntegerQ] instead.
                $endgroup$
                – Jerry
                1 hour ago












              • $begingroup$
                Good point, I added the pattern after posting...
                $endgroup$
                – Henrik Schumacher
                1 hour ago











              Your Answer





              StackExchange.ifUsing("editor", function () {
              return StackExchange.using("mathjaxEditing", function () {
              StackExchange.MarkdownEditor.creationCallbacks.add(function (editor, postfix) {
              StackExchange.mathjaxEditing.prepareWmdForMathJax(editor, postfix, [["$", "$"], ["\\(","\\)"]]);
              });
              });
              }, "mathjax-editing");

              StackExchange.ready(function() {
              var channelOptions = {
              tags: "".split(" "),
              id: "387"
              };
              initTagRenderer("".split(" "), "".split(" "), channelOptions);

              StackExchange.using("externalEditor", function() {
              // Have to fire editor after snippets, if snippets enabled
              if (StackExchange.settings.snippets.snippetsEnabled) {
              StackExchange.using("snippets", function() {
              createEditor();
              });
              }
              else {
              createEditor();
              }
              });

              function createEditor() {
              StackExchange.prepareEditor({
              heartbeatType: 'answer',
              autoActivateHeartbeat: false,
              convertImagesToLinks: false,
              noModals: true,
              showLowRepImageUploadWarning: true,
              reputationToPostImages: null,
              bindNavPrevention: true,
              postfix: "",
              imageUploader: {
              brandingHtml: "Powered by u003ca class="icon-imgur-white" href="https://imgur.com/"u003eu003c/au003e",
              contentPolicyHtml: "User contributions licensed under u003ca href="https://creativecommons.org/licenses/by-sa/3.0/"u003ecc by-sa 3.0 with attribution requiredu003c/au003e u003ca href="https://stackoverflow.com/legal/content-policy"u003e(content policy)u003c/au003e",
              allowUrls: true
              },
              onDemand: true,
              discardSelector: ".discard-answer"
              ,immediatelyShowMarkdownHelp:true
              });


              }
              });














              draft saved

              draft discarded


















              StackExchange.ready(
              function () {
              StackExchange.openid.initPostLogin('.new-post-login', 'https%3a%2f%2fmathematica.stackexchange.com%2fquestions%2f189729%2fgenerate-this-sequence-more-efficiently%23new-answer', 'question_page');
              }
              );

              Post as a guest















              Required, but never shown

























              3 Answers
              3






              active

              oldest

              votes








              3 Answers
              3






              active

              oldest

              votes









              active

              oldest

              votes






              active

              oldest

              votes









              6












              $begingroup$

              ClearAll[f]
              f[n_Integer] := Join[Range[1, #, 2], Reverse[Range[2, #, 2]]] & /@ Range[2, n];
              TeXForm @ MatrixForm @ f[8]



              $left(
              begin{array}{c}
              {1,2} \
              {1,3,2} \
              {1,3,4,2} \
              {1,3,5,4,2} \
              {1,3,5,6,4,2} \
              {1,3,5,7,6,4,2} \
              {1,3,5,7,8,6,4,2} \
              end{array}
              right)$




              Also



              ClearAll[f2, f3]
              f2[n_Integer] := SortBy[Range@#, {EvenQ, -# (-1 )^Mod[#, 2] &}] & /@ Range[2, n]
              f3[n_] := Ordering[Transpose[{-Mod[#, 2], -# (-1 )^Mod[#, 2]} &@Range[#]]] & /@ Range[2, n]

              f[8] == f2[8] == f3[8]



              True







              share|improve this answer











              $endgroup$


















                6












                $begingroup$

                ClearAll[f]
                f[n_Integer] := Join[Range[1, #, 2], Reverse[Range[2, #, 2]]] & /@ Range[2, n];
                TeXForm @ MatrixForm @ f[8]



                $left(
                begin{array}{c}
                {1,2} \
                {1,3,2} \
                {1,3,4,2} \
                {1,3,5,4,2} \
                {1,3,5,6,4,2} \
                {1,3,5,7,6,4,2} \
                {1,3,5,7,8,6,4,2} \
                end{array}
                right)$




                Also



                ClearAll[f2, f3]
                f2[n_Integer] := SortBy[Range@#, {EvenQ, -# (-1 )^Mod[#, 2] &}] & /@ Range[2, n]
                f3[n_] := Ordering[Transpose[{-Mod[#, 2], -# (-1 )^Mod[#, 2]} &@Range[#]]] & /@ Range[2, n]

                f[8] == f2[8] == f3[8]



                True







                share|improve this answer











                $endgroup$
















                  6












                  6








                  6





                  $begingroup$

                  ClearAll[f]
                  f[n_Integer] := Join[Range[1, #, 2], Reverse[Range[2, #, 2]]] & /@ Range[2, n];
                  TeXForm @ MatrixForm @ f[8]



                  $left(
                  begin{array}{c}
                  {1,2} \
                  {1,3,2} \
                  {1,3,4,2} \
                  {1,3,5,4,2} \
                  {1,3,5,6,4,2} \
                  {1,3,5,7,6,4,2} \
                  {1,3,5,7,8,6,4,2} \
                  end{array}
                  right)$




                  Also



                  ClearAll[f2, f3]
                  f2[n_Integer] := SortBy[Range@#, {EvenQ, -# (-1 )^Mod[#, 2] &}] & /@ Range[2, n]
                  f3[n_] := Ordering[Transpose[{-Mod[#, 2], -# (-1 )^Mod[#, 2]} &@Range[#]]] & /@ Range[2, n]

                  f[8] == f2[8] == f3[8]



                  True







                  share|improve this answer











                  $endgroup$



                  ClearAll[f]
                  f[n_Integer] := Join[Range[1, #, 2], Reverse[Range[2, #, 2]]] & /@ Range[2, n];
                  TeXForm @ MatrixForm @ f[8]



                  $left(
                  begin{array}{c}
                  {1,2} \
                  {1,3,2} \
                  {1,3,4,2} \
                  {1,3,5,4,2} \
                  {1,3,5,6,4,2} \
                  {1,3,5,7,6,4,2} \
                  {1,3,5,7,8,6,4,2} \
                  end{array}
                  right)$




                  Also



                  ClearAll[f2, f3]
                  f2[n_Integer] := SortBy[Range@#, {EvenQ, -# (-1 )^Mod[#, 2] &}] & /@ Range[2, n]
                  f3[n_] := Ordering[Transpose[{-Mod[#, 2], -# (-1 )^Mod[#, 2]} &@Range[#]]] & /@ Range[2, n]

                  f[8] == f2[8] == f3[8]



                  True








                  share|improve this answer














                  share|improve this answer



                  share|improve this answer








                  edited 3 hours ago

























                  answered 4 hours ago









                  kglrkglr

                  179k9199410




                  179k9199410























                      5












                      $begingroup$

                         fGetList[n_]:= (Select[Range[#], OddQ]~Join~Reverse@Select[Range[#], EvenQ]) & /@Range[n] // Rest

                      fGetList[10] // MatrixForm // TeXForm


                      $
                      left(
                      begin{array}{c}
                      {1,2} \
                      {1,3,2} \
                      {1,3,4,2} \
                      {1,3,5,4,2} \
                      {1,3,5,6,4,2} \
                      {1,3,5,7,6,4,2} \
                      {1,3,5,7,8,6,4,2} \
                      {1,3,5,7,9,8,6,4,2} \
                      {1,3,5,7,9,10,8,6,4,2} \
                      end{array}
                      right)$



                      another version



                      fGetList2[n_?IntegerQ] := 
                      Flatten@MapAt[Reverse, GatherBy[Range[#], OddQ], 2] & /@ Range[2, n]

                      fGetList2[10] // MatrixForm // TeXForm


                      $left(
                      begin{array}{c}
                      {1,2} \
                      {1,3,2} \
                      {1,3,4,2} \
                      {1,3,5,4,2} \
                      {1,3,5,6,4,2} \
                      {1,3,5,7,6,4,2} \
                      {1,3,5,7,8,6,4,2} \
                      {1,3,5,7,9,8,6,4,2} \
                      {1,3,5,7,9,10,8,6,4,2} \
                      end{array}
                      right)$






                      share|improve this answer











                      $endgroup$


















                        5












                        $begingroup$

                           fGetList[n_]:= (Select[Range[#], OddQ]~Join~Reverse@Select[Range[#], EvenQ]) & /@Range[n] // Rest

                        fGetList[10] // MatrixForm // TeXForm


                        $
                        left(
                        begin{array}{c}
                        {1,2} \
                        {1,3,2} \
                        {1,3,4,2} \
                        {1,3,5,4,2} \
                        {1,3,5,6,4,2} \
                        {1,3,5,7,6,4,2} \
                        {1,3,5,7,8,6,4,2} \
                        {1,3,5,7,9,8,6,4,2} \
                        {1,3,5,7,9,10,8,6,4,2} \
                        end{array}
                        right)$



                        another version



                        fGetList2[n_?IntegerQ] := 
                        Flatten@MapAt[Reverse, GatherBy[Range[#], OddQ], 2] & /@ Range[2, n]

                        fGetList2[10] // MatrixForm // TeXForm


                        $left(
                        begin{array}{c}
                        {1,2} \
                        {1,3,2} \
                        {1,3,4,2} \
                        {1,3,5,4,2} \
                        {1,3,5,6,4,2} \
                        {1,3,5,7,6,4,2} \
                        {1,3,5,7,8,6,4,2} \
                        {1,3,5,7,9,8,6,4,2} \
                        {1,3,5,7,9,10,8,6,4,2} \
                        end{array}
                        right)$






                        share|improve this answer











                        $endgroup$
















                          5












                          5








                          5





                          $begingroup$

                             fGetList[n_]:= (Select[Range[#], OddQ]~Join~Reverse@Select[Range[#], EvenQ]) & /@Range[n] // Rest

                          fGetList[10] // MatrixForm // TeXForm


                          $
                          left(
                          begin{array}{c}
                          {1,2} \
                          {1,3,2} \
                          {1,3,4,2} \
                          {1,3,5,4,2} \
                          {1,3,5,6,4,2} \
                          {1,3,5,7,6,4,2} \
                          {1,3,5,7,8,6,4,2} \
                          {1,3,5,7,9,8,6,4,2} \
                          {1,3,5,7,9,10,8,6,4,2} \
                          end{array}
                          right)$



                          another version



                          fGetList2[n_?IntegerQ] := 
                          Flatten@MapAt[Reverse, GatherBy[Range[#], OddQ], 2] & /@ Range[2, n]

                          fGetList2[10] // MatrixForm // TeXForm


                          $left(
                          begin{array}{c}
                          {1,2} \
                          {1,3,2} \
                          {1,3,4,2} \
                          {1,3,5,4,2} \
                          {1,3,5,6,4,2} \
                          {1,3,5,7,6,4,2} \
                          {1,3,5,7,8,6,4,2} \
                          {1,3,5,7,9,8,6,4,2} \
                          {1,3,5,7,9,10,8,6,4,2} \
                          end{array}
                          right)$






                          share|improve this answer











                          $endgroup$



                             fGetList[n_]:= (Select[Range[#], OddQ]~Join~Reverse@Select[Range[#], EvenQ]) & /@Range[n] // Rest

                          fGetList[10] // MatrixForm // TeXForm


                          $
                          left(
                          begin{array}{c}
                          {1,2} \
                          {1,3,2} \
                          {1,3,4,2} \
                          {1,3,5,4,2} \
                          {1,3,5,6,4,2} \
                          {1,3,5,7,6,4,2} \
                          {1,3,5,7,8,6,4,2} \
                          {1,3,5,7,9,8,6,4,2} \
                          {1,3,5,7,9,10,8,6,4,2} \
                          end{array}
                          right)$



                          another version



                          fGetList2[n_?IntegerQ] := 
                          Flatten@MapAt[Reverse, GatherBy[Range[#], OddQ], 2] & /@ Range[2, n]

                          fGetList2[10] // MatrixForm // TeXForm


                          $left(
                          begin{array}{c}
                          {1,2} \
                          {1,3,2} \
                          {1,3,4,2} \
                          {1,3,5,4,2} \
                          {1,3,5,6,4,2} \
                          {1,3,5,7,6,4,2} \
                          {1,3,5,7,8,6,4,2} \
                          {1,3,5,7,9,8,6,4,2} \
                          {1,3,5,7,9,10,8,6,4,2} \
                          end{array}
                          right)$







                          share|improve this answer














                          share|improve this answer



                          share|improve this answer








                          edited 3 hours ago

























                          answered 4 hours ago









                          JerryJerry

                          999112




                          999112























                              2












                              $begingroup$

                              cg = Compile[{{a, _Integer, 1}, {b, _Integer, 1}, {i, _Integer}},
                              Join[a[[1 ;; Quotient[i + 1, 2]]], b[[-Quotient[i, 2] ;; -1]]],
                              CompilationTarget -> "WVM",
                              RuntimeAttributes -> {Listable},
                              Parallelization -> True
                              ];
                              g[n_Integer] := cg[Range[1, n + 1, 2], Range[n + Mod[n, 2], 2, -2], Range[2, n + 1]];





                              share|improve this answer











                              $endgroup$













                              • $begingroup$
                                there's a little bug with g[n_?Integer], use g[n_Integer] or g[n_?IntegerQ] instead.
                                $endgroup$
                                – Jerry
                                1 hour ago












                              • $begingroup$
                                Good point, I added the pattern after posting...
                                $endgroup$
                                – Henrik Schumacher
                                1 hour ago
















                              2












                              $begingroup$

                              cg = Compile[{{a, _Integer, 1}, {b, _Integer, 1}, {i, _Integer}},
                              Join[a[[1 ;; Quotient[i + 1, 2]]], b[[-Quotient[i, 2] ;; -1]]],
                              CompilationTarget -> "WVM",
                              RuntimeAttributes -> {Listable},
                              Parallelization -> True
                              ];
                              g[n_Integer] := cg[Range[1, n + 1, 2], Range[n + Mod[n, 2], 2, -2], Range[2, n + 1]];





                              share|improve this answer











                              $endgroup$













                              • $begingroup$
                                there's a little bug with g[n_?Integer], use g[n_Integer] or g[n_?IntegerQ] instead.
                                $endgroup$
                                – Jerry
                                1 hour ago












                              • $begingroup$
                                Good point, I added the pattern after posting...
                                $endgroup$
                                – Henrik Schumacher
                                1 hour ago














                              2












                              2








                              2





                              $begingroup$

                              cg = Compile[{{a, _Integer, 1}, {b, _Integer, 1}, {i, _Integer}},
                              Join[a[[1 ;; Quotient[i + 1, 2]]], b[[-Quotient[i, 2] ;; -1]]],
                              CompilationTarget -> "WVM",
                              RuntimeAttributes -> {Listable},
                              Parallelization -> True
                              ];
                              g[n_Integer] := cg[Range[1, n + 1, 2], Range[n + Mod[n, 2], 2, -2], Range[2, n + 1]];





                              share|improve this answer











                              $endgroup$



                              cg = Compile[{{a, _Integer, 1}, {b, _Integer, 1}, {i, _Integer}},
                              Join[a[[1 ;; Quotient[i + 1, 2]]], b[[-Quotient[i, 2] ;; -1]]],
                              CompilationTarget -> "WVM",
                              RuntimeAttributes -> {Listable},
                              Parallelization -> True
                              ];
                              g[n_Integer] := cg[Range[1, n + 1, 2], Range[n + Mod[n, 2], 2, -2], Range[2, n + 1]];






                              share|improve this answer














                              share|improve this answer



                              share|improve this answer








                              edited 1 hour ago

























                              answered 2 hours ago









                              Henrik SchumacherHenrik Schumacher

                              50.5k469144




                              50.5k469144












                              • $begingroup$
                                there's a little bug with g[n_?Integer], use g[n_Integer] or g[n_?IntegerQ] instead.
                                $endgroup$
                                – Jerry
                                1 hour ago












                              • $begingroup$
                                Good point, I added the pattern after posting...
                                $endgroup$
                                – Henrik Schumacher
                                1 hour ago


















                              • $begingroup$
                                there's a little bug with g[n_?Integer], use g[n_Integer] or g[n_?IntegerQ] instead.
                                $endgroup$
                                – Jerry
                                1 hour ago












                              • $begingroup$
                                Good point, I added the pattern after posting...
                                $endgroup$
                                – Henrik Schumacher
                                1 hour ago
















                              $begingroup$
                              there's a little bug with g[n_?Integer], use g[n_Integer] or g[n_?IntegerQ] instead.
                              $endgroup$
                              – Jerry
                              1 hour ago






                              $begingroup$
                              there's a little bug with g[n_?Integer], use g[n_Integer] or g[n_?IntegerQ] instead.
                              $endgroup$
                              – Jerry
                              1 hour ago














                              $begingroup$
                              Good point, I added the pattern after posting...
                              $endgroup$
                              – Henrik Schumacher
                              1 hour ago




                              $begingroup$
                              Good point, I added the pattern after posting...
                              $endgroup$
                              – Henrik Schumacher
                              1 hour ago


















                              draft saved

                              draft discarded




















































                              Thanks for contributing an answer to Mathematica Stack Exchange!


                              • Please be sure to answer the question. Provide details and share your research!

                              But avoid



                              • Asking for help, clarification, or responding to other answers.

                              • Making statements based on opinion; back them up with references or personal experience.


                              Use MathJax to format equations. MathJax reference.


                              To learn more, see our tips on writing great answers.




                              draft saved


                              draft discarded














                              StackExchange.ready(
                              function () {
                              StackExchange.openid.initPostLogin('.new-post-login', 'https%3a%2f%2fmathematica.stackexchange.com%2fquestions%2f189729%2fgenerate-this-sequence-more-efficiently%23new-answer', 'question_page');
                              }
                              );

                              Post as a guest















                              Required, but never shown





















































                              Required, but never shown














                              Required, but never shown












                              Required, but never shown







                              Required, but never shown

































                              Required, but never shown














                              Required, but never shown












                              Required, but never shown







                              Required, but never shown







                              Popular posts from this blog

                              GameSpot

                              connect to host localhost port 22: Connection refused

                              Getting a Wifi WPA2 wifi connection